GERIATRICS Flashcards
Questions 1 and 2 pertain to the following case.
An 85-year-old man presents to the primary care clinic 1
month after the death of his spouse. His medical history
is significant for hypertension, hyperlipidemia, benign
prostatic hyperplasia (BPH), and major depressive disorder.
His current medications include metoprolol XL
25 mg daily, atorvastatin 20 mg daily, tamsulosin 0.4
mg daily, diazepam 5 mg at bedtime as needed for sleep,
and escitalopram 10 mg daily. His daughter reports that
he has been more lethargic and unsteady walking during
the past 3 days. The patient reports trouble sleeping,
necessitating the use of diazepam every night this past
week. His blood pressure is 135/72 mm Hg and heart
rate is 76 beats/minute. Urinalysis is unremarkable, thyroid-
stimulating hormone (TSH) is within the reference
range, and Geriatric Depression Scale score is 6/15.
1. Which medication is most contributing to this
patient’s lethargy and confusion?
A. Diazepam.
B. Metoprolol.
C. Atorvastatin.
D. Escitalopram.
- Answer: A
Diazepam is a long-acting benzodiazepine that can
accumulate in older patients, resulting in excessive lethargy,
sedation, and unsteady gait, and the patient admits
taking it every night during the past week (Answer A is
correct). A worsening of the patient’s depression is evident
with the recent bereavement; however, that would
not explain the unsteady gait (Answer D is incorrect).
Although metoprolol can cause lethargy, the patient’s
current BP and metoprolol’s beta-1 selective nature
would make metoprolol the unlikely cause of this new
change in lethargy and unsteady gait (Answer B is
incorrect). Atorvastatin is not a common cause of lethargy
and confusion (Answer C is incorrect).
Questions 1 and 2 pertain to the following case.
An 85-year-old man presents to the primary care clinic 1
month after the death of his spouse. His medical history
is significant for hypertension, hyperlipidemia, benign
prostatic hyperplasia (BPH), and major depressive disorder.
His current medications include metoprolol XL
25 mg daily, atorvastatin 20 mg daily, tamsulosin 0.4
mg daily, diazepam 5 mg at bedtime as needed for sleep,
and escitalopram 10 mg daily. His daughter reports that
he has been more lethargic and unsteady walking during
the past 3 days. The patient reports trouble sleeping,
necessitating the use of diazepam every night this past
week. His blood pressure is 135/72 mm Hg and heart
rate is 76 beats/minute. Urinalysis is unremarkable, thyroid-
stimulating hormone (TSH) is within the reference
range, and Geriatric Depression Scale score is 6/15.
- Which age-related change in pharmacokinetics
most likely underlies this patient’s medication-related
problem?
A. Delayed oral absorption.
B. Decreased renal excretion.
C. Slowed metabolism in the liver.
D. Decreased volume of distribution.
- Answer: C
Due to slowed liver metabolism in older adults, accumulation
of multiple active metabolites can be expected,
leading to increased risk of sedation and risk of falls
(Answer C is correct). In older patients, the volume
of distribution of lipidsoluble drugs such as diazepam
is increased, not decreased (Answer D is incorrect).
Diazepam is converted to active metabolites temazepam
and further to oxazepam through CYP3A4. Oral
absorption is not significantly altered in older adults for
chronic medications (Answer A is incorrect). Decreased
renal excretion is likely but is not a significant contributor
in this patient, given the drugs on his medication list
(Answer B is incorrect).
Questions 3 and 4 pertain to the following case.
A 76-year-old woman was recently admitted to a longterm
care facility for rehabilitation after several falls at
home. Her medical history is significant for hypertension,
hypothyroidism, Alzheimer disease (AD), hyperlipidemia,
and osteoarthritis (OA) of the knee. She takes
metoprolol succinate 50 mg daily, levothyroxine 75 mcg
daily, atorvastatin 10 mg daily, and donepezil 10 mg
daily. Her blood pressure is 126/80 mm Hg and heart
rate is 66 beats/minute. Basic metabolic panel results
are all within reference ranges; 25-hydroxyvitamin D concentration is 20 ng/mL, TSH is 1.89 mU/L, total
cholesterol is 180 mg/dL, low-density lipoprotein cholesterol
is 140 mg/dL, high-density lipoprotein cholesterol
is 35 mg/dL, and triglycerides is 176 mg/dL.
Her Mini–Mental State Examination (MMSE) score is
16/30, and her Geriatric Depression Scale score is 2/15.
3. Which recommendation would be most appropriate
to reduce this patient’s risk of falls?
A. Initiate memantine 5 mg daily.
B. Initiate vitamin D 1000 units daily.
C. Initiate aducanumab 1 mg/kg infusion every
4 weeks.
D. Initiate calcium carbonate 500 mg twice daily
- Answer: B
Adding vitamin D to this resident’s regimen, given
her deficient serum concentrations, may help reduce
falls (Answer B is correct). Insufficient information
is provided to determine the need to add memantine
at this time (Answer A is incorrect). Aducanumab is
currently only approved for MCI or mild AD, based on
the patient’s MMSE of 16/30, the patient would be considered
to have moderate dementia and not qualify for
aducanumab (Answer C is incorrect). Adding calcium
carbonate might help reduce fractures but would not
reduce fall risk (Answer D is incorrect).
Questions 3 and 4 pertain to the following case.
A 76-year-old woman was recently admitted to a longterm
care facility for rehabilitation after several falls at
home. Her medical history is significant for hypertension,
hypothyroidism, Alzheimer disease (AD), hyperlipidemia,
and osteoarthritis (OA) of the knee. She takes
metoprolol succinate 50 mg daily, levothyroxine 75 mcg
daily, atorvastatin 10 mg daily, and donepezil 10 mg
daily. Her blood pressure is 126/80 mm Hg and heart
rate is 66 beats/minute. Basic metabolic panel results
are all within reference ranges; 25-hydroxyvitamin D concentration is 20 ng/mL, TSH is 1.89 mU/L, total
cholesterol is 180 mg/dL, low-density lipoprotein cholesterol
is 140 mg/dL, high-density lipoprotein cholesterol
is 35 mg/dL, and triglycerides is 176 mg/dL.
Her Mini–Mental State Examination (MMSE) score is
16/30, and her Geriatric Depression Scale score is 2/15.
- Which would be most appropriate for the patient’s
osteoarthritic knee pain?
A. Ibuprofen 200 mg four times daily.
B. Acetaminophen 650 mg three times daily.
C. Tramadol 50 mg three times daily as needed
for pain.
D. Diclofenac 1% topical gel 4 g applied to knee
four times daily.
- Answer: B
NSAIDs are recommended first-line for patients with
OA. For patients greater than 75 years old with OA of the knee, topical NSAIDs are preferred over systemic
NSAIDs due to lower risk of systemic side effects
(Answer D is correct; Answer A is incorrect). A trial
of acetaminophen at doses less than 3 g/day is reasonable
for frail patients with OA of the knee after a trial
of a topical NSAID has failed (Answer B is incorrect).
Tramadol at doses less than 200 mg/day would be a
reasonable alternative when more conservative medications
have failed a trial of 1–2 weeks (Answer C is
incorrect).
Questions 5–7 pertain to the following case.
An 80-year-old woman presents to your clinic accompanied
by her daughter, who no longer feels comfortable
leaving her mother alone because of her mother’s
“increasing forgetfulness.” The patient’s medical history
is significant for type 2 diabetes, hypertension,
coronary artery disease, congestive heart failure, and
OA. She takes the following medications: acetaminophen
650 mg every 6 hours as needed for pain, lisinopril
20 mg daily, furosemide 20 mg daily, potassium
chloride 20 mEq daily, carvedilol 12.5 mg twice daily,
and glipizide 5 mg daily. Her MMSE score is 18/30.
Blood tests obtained last week showed a normal basic
metabolic panel, except for a fasting plasma glucose
reading of 65 mg/dL. Her hemoglobin A1C (A1C) is
5.6%. A urinalysis is unremarkable. No nutritional
deficiencies are noted. The patient’s blood pressure is
130/80 mm Hg and heart rate is 60 beats/minute. She
receives a diagnosis of AD.
- Which initial intervention would be most appropriate
to help with this patient’s cognitive function?
A. Donepezil 10 mg daily.
B. Galantamine extended release (ER) 24 mg
daily.
C. Memantine 10 mg twice daily.
D. Rivastigmine patch 4.6 mg daily
- Answer: D
All CIs have similar efficacy. The rivastigmine transdermal
patch is better tolerated than oral rivastigmine,
and 4.6 mg is the appropriate initial starting dose of
the transdermal patch (Answer D is correct). Although
donepezil tends to be the best tolerated CI, doses of
cholinesterase medications should be titrated slowly
to prevent GI upset. The initial donepezil dose is 5 mg
daily at bedtime, and for galantamine ER, the dose
is 8 mg once daily (Answers A and B are incorrect).
Memantine has no beneficial effect in maintaining cognitive
function, as measured by MMSE scores (Answer
C is incorrect).
Questions 5–7 pertain to the following case.
An 80-year-old woman presents to your clinic accompanied
by her daughter, who no longer feels comfortable
leaving her mother alone because of her mother’s
“increasing forgetfulness.” The patient’s medical history
is significant for type 2 diabetes, hypertension,
coronary artery disease, congestive heart failure, and
OA. She takes the following medications: acetaminophen
650 mg every 6 hours as needed for pain, lisinopril
20 mg daily, furosemide 20 mg daily, potassium
chloride 20 mEq daily, carvedilol 12.5 mg twice daily,
and glipizide 5 mg daily. Her MMSE score is 18/30.
Blood tests obtained last week showed a normal basic
metabolic panel, except for a fasting plasma glucose
reading of 65 mg/dL. Her hemoglobin A1C (A1C) is
5.6%. A urinalysis is unremarkable. No nutritional
deficiencies are noted. The patient’s blood pressure is
130/80 mm Hg and heart rate is 60 beats/minute. She
receives a diagnosis of AD.
- Which intervention would be most appropriate to
prevent an adverse drug reaction?
A. Discontinue glipizide.
B. Discontinue lisinopril.
C. Reduce carvedilol to 6.25 mg twice daily.
D. Reduce potassium chloride to 10 mEq daily.
- Answer: A
This patient’s current fasting blood glucose of 65 mg/
dL and A1C of 5.6% should prompt the pharmacist to
request glipizide discontinuation (Answer A is correct).
According to the American Diabetes Association’s
Standards of Medical Care in Diabetes guidelines, older
adults with several chronic illnesses, cognitive impairment,
or functional dependence should have less stringent
glycemic goals, such as A1C less than 8.0%–8.5%.
The goals of therapy are to prevent hypoglycemia in
older patients at greatest risk of this adverse drug reaction.
There is no rationale for reducing the carvedilol
dose, and given her normal basic metabolic panel and
blood pressure, reducing potassium chloride or discontinuing
lisinopril is not indicated at this time (Answers
B–D are incorrect).
Questions 5–7 pertain to the following case.
An 80-year-old woman presents to your clinic accompanied
by her daughter, who no longer feels comfortable
leaving her mother alone because of her mother’s
“increasing forgetfulness.” The patient’s medical history
is significant for type 2 diabetes, hypertension,
coronary artery disease, congestive heart failure, and
OA. She takes the following medications: acetaminophen
650 mg every 6 hours as needed for pain, lisinopril
20 mg daily, furosemide 20 mg daily, potassium
chloride 20 mEq daily, carvedilol 12.5 mg twice daily,
and glipizide 5 mg daily. Her MMSE score is 18/30.
Blood tests obtained last week showed a normal basic
metabolic panel, except for a fasting plasma glucose
reading of 65 mg/dL. Her hemoglobin A1C (A1C) is
5.6%. A urinalysis is unremarkable. No nutritional
deficiencies are noted. The patient’s blood pressure is
130/80 mm Hg and heart rate is 60 beats/minute. She
receives a diagnosis of AD.
- One year later, the patient returns to the clinic.
She has moved in with her daughter. Lately, she
wanders around the house continuously. She often
changes clothes, cries out, and asks repetitive questions.
Her current medication regimen includes
rivastigmine 9.5 transdermal patch daily, which
she has been taking for the past 6 months. Which
would be most appropriate for this patient’s new
behavioral symptoms?
A. Initiate olanzapine 5 mg daily.
B. Initiate risperidone 0.5 mg twice daily.
C. Initiate pimavanserin 34 mg daily.
D. Change acetaminophen to 650 mg every 6
hours around-the-clock.
- Answer: D
Identifying reversible or underlying causes, such as
pain or constipation, should be completed first when
a patient experiences new behavioral changes. Given this patient’s history of OA and advancing dementia,
they may not be able to accurately communicate their
pain. Changing the acetaminophen from as needed to
scheduled, should be the first intervention to rule out
an underlying cause to this new behavior (Answer D
is correct). The off-label use of atypical antipsychotic
medications in patients with behavioral symptoms of
dementia should be reserved for patients who pose a
danger to themselves or others or experience hallucinations
or delusions that are stressful to them (Answers
A and B are incorrect). Currently there is no evidence
supporting the use of pimavenserin in AD, and it should
only be reserved for patients with Parkinson disease
psychosis (Answer C is incorrect).
- An 80-year-old woman had a total right knee
replacement 3 days ago after conservative strategies
for OA failed. Her medical history is significant
for hypothyroidism, osteoporosis, OA, and
hyperlipidemia. Her current medications include
simvastatin 20 mg daily, risedronate 35 mg weekly,
levothyroxine 75 mcg daily, and oxycodone/
acetaminophen 5/325 mg 1 tablet every 4 hours as
needed for moderate pain. She is in the hospital
preparing for discharge. As the pharmacist is counseling
the patient on her discharge medication, the
patient reports a new onset of “losing her water”
the day before and again overnight. Which intervention
would be most appropriate for this patient?
A. Urinalysis.
B. Pelvic floor exercises.
C. Mirabegron 25 mg daily.
D. Duloxetine 20 mg daily.
- Answer: A
Any new symptom of UI in an older adult should be
thoroughly evaluated to determine whether there is a
reversible cause. Infection, or the “I” in the mnemonic
DRIP, may be the cause of the new symptoms in this
patient. Urinalysis would be the most appropriate
intervention for this reversible cause of incontinence
(Answer A is correct). Mirabegron is a reasonable
option for urge incontinence or over-active bladder
once reversible causes have been ruled out (Answer
C is incorrect). Duloxetine has been used off-label for
stress incontinence (Answer D is incorrect). Pelvic floor
muscle exercises or Kegel exercises should be firstline
therapy for stress, urge, or mixed incontinence in
women (Answer B is incorrect).
Questions 9 and 10 pertain to the following case.
A 69-year-old man is admitted to the hospital after a
motorcycle collision. He had serious injuries resulting
in a left leg above-the-knee amputation and has undergone
several surgical procedures and rehabilitation
in the past 2 weeks. His current medications include
tamsulosin 0.4 mg daily, atenolol 25 mg daily, amlodipine
10 mg daily, senna/docusate 8.6/50 mg twice daily,
oxycodone controlled release 10 mg every 12 hours,
and hydromorphone 4 mg every 3 hours as needed for
breakthrough pain (uses 1–2 daily). His blood pressure
is 155/88 mm Hg, heart rate is 84 beats/minute,
and postvoid residual (PVR) volume is 400 mL after
voiding 110 mL. His chronic medical conditions are
unremarkable except for hypertension, BPH, and gastroesophageal
reflux disease.
9. Which intervention would be most appropriate for
this patient?
A. Change tamsulosin to alfuzosin 10 mg once
daily.
B. Increase atenolol to 50 mg daily.
C. Change tamsulosin to doxazosin 1 mg daily.
D. Reduce hydromorphone to 2 mg every 3 hours
as needed for breakthrough pain.
- Answer: C
In this patient with comorbid conditions of hypertension
and BPH, the choice of α-blockers is based on the adverse
effect profiles. This patient has an elevated PVR volume,
so changing tamsulosin to doxazosin might reduce both
blood pressure and urinary retention; merely changing to
another selective α-blocker might not provide adequate
relief of both conditions. (Answer C is correct; Answer A
is incorrect). Increasing the atenolol dose would address
only the increased blood pressure, without affecting the
current problem of acute urinary retention (Answer B
is incorrect). The patient is receiving moderate doses of
controlled-release opioid, so reducing the hydromorphone
dose for breakthrough pain is unlikely to help
reduce the obstruction that may be worsened by the narcotics
(Answer D is incorrect).
Questions 9 and 10 pertain to the following case.
A 69-year-old man is admitted to the hospital after a
motorcycle collision. He had serious injuries resulting
in a left leg above-the-knee amputation and has undergone
several surgical procedures and rehabilitation
in the past 2 weeks. His current medications include
tamsulosin 0.4 mg daily, atenolol 25 mg daily, amlodipine
10 mg daily, senna/docusate 8.6/50 mg twice daily,
oxycodone controlled release 10 mg every 12 hours,
and hydromorphone 4 mg every 3 hours as needed for
breakthrough pain (uses 1–2 daily). His blood pressure
is 155/88 mm Hg, heart rate is 84 beats/minute,
and postvoid residual (PVR) volume is 400 mL after
voiding 110 mL. His chronic medical conditions are
unremarkable except for hypertension, BPH, and gastroesophageal
reflux disease.
- One year later, the patient has returned for
complaints related to his BPH. His blood pressure
is 118/74 mm Hg, heart rate 78 beats/minute, and
PVR volume is 220 mL after voiding 150 mL.
Current medications include doxazosin 4 mg daily,
atenolol 25 mg daily, and amlodipine 10 mg daily.
What is the most appropriate intervention for this
patient?
A. Initiate tadalafil 5 mg daily.
B. Initiate finasteride 5 mg daily.
C. Initiate alfuzosin 10 mg daily.
D. Initiate saw palmetto supplement daily 40 mg
injected into affected joint.
- One year later, the patient has returned for
- Answer: C
Due to continued elevated PVR volume with reported
symptoms, therapy combination is recommended for
this patient. Doxazosin plus finasteride is the most
extensively studied (Answer B is correct). Tamsulosin
plus dutasteride is also an extensively studied combination
in BPH. Tadalafil use with doxazosin and other
nonselective alpha1 blockers is not recommended due
to increased risk of hypotension (Answer A is incorrect).
Alfuzosin is a selective alpha1 blocker and should
not be used in combination with other alpha1 blockers
(Answer C is incorrect). Currently there is not enough
evidence to support the efficacy of saw palmetto
(Answer D is incorrect).
- A 72-year-old woman (height 66 inches, weight
82 kg) whose medical history is significant for
rheumatoid arthritis (RA), type 2 diabetes, gastroesophageal
reflux disease, and hypothyroidism
presents to the clinic with inflammation of
the joints of the hands and stiffness lasting 1–2
hours in the morning. She is a smoker. Her current
medications include pantoprazole 40 mg daily,
metformin 850 mg twice daily, levothyroxine 100
mcg daily, folic acid 1 mg daily, methotrexate 12.5
mg weekly, naproxen 500 mg twice daily, calcium
600 mg twice daily, and vitamin D 1000 units twice
daily. Her laboratory tests show a negative rheumatoid
factor (RF) but positive anti–cyclic citrullinated
peptides. The physician determines that
this is a flare of moderate disease. Which would be
the most appropriate intervention for maintenance
treatment of this patient’s RA?
A. Change naproxen to prednisone 20 mg daily.
B. Change methotrexate to 25 mg intramuscularly.
C. Change methotrexate to leflunomide 20 mg
daily.
D. Add sulfasalazine 500 mg twice daily and
hydroxychloroquine 400 mg daily.
- Answer: D
In patients with recurring RA symptoms, moderate
disease activity, and the presence of a poor prognostic
factor (anti–cyclic citrullinated peptides), adding sulfasalazine
and hydroxychloroquine to methotrexate
follows guidelines from the 2015 American College
of Rheumatology recommendations update for the
treatment of RA (Answer D is correct). Specifically,
these guidelines recommend either double- or triplecombination
DMARD therapy for patients with an inadequate
response to methotrexate. Prednisone may be
used as bridge therapy, but continued therapy may not be
supported by a risk-benefit analysis (Answer A is incorrect).
Changing methotrexate from the oral route to the
intramuscular route would offer no significant benefit in
this case (Answer B is incorrect). Similarly, changing
methotrexate to monotherapy with leflunomide would
provide no significant benefits (Answer C is incorrect).
- A 66-year-old man is initiated on allopurinol
100 mg once daily and naproxen 500 mg twice
daily for the treatment of an acute gout flare. On
diagnosis, no tophi were present, serum urate was
10.9 mg/dL, and GFR was 78 mL/minute/1.73 m2.
His allopurinol dose is increased 2 weeks after initial
presentation to 200 mg, and at his 1-month follow-
up, the allopurinol dose was further increased
to 300 mg once daily. Naproxen was discontinued
because his symptoms had resolved; serum
urate was 8.7 mg/dL and GFR was 84 mL/minute/
1.73 m2. He is seen 2 weeks later for an emergency
follow-up because he has developed a new
rash; serum urate is 7.4 mg/dL and GFR is 72 mL/
minute/1.73 m2. Which is best for managing his
gout at this time?
A. Increasing allopurinol to 400 mg daily.
B. Reinitiating naproxen 500 mg twice daily.
C. Changing allopurinol to febuxostat 40 mg
daily.
D. Changing allopurinol to pegloticase 8 mg
intravenously every 2 weeks.
- Answer: C
Trial of a different XAO (e.g., febuxostat) should be
initiated in the setting of intolerance or adverse effects
before a uricosuric agent is tried (Answer C is correct).
The allopurinol dose should not be increased at this
time because of the presence of a new-onset rash. This
may be an adverse effect associated with allopurinol;
therefore, allopurinol should be discontinued (Answer
A is incorrect). There is no acute need to restart NSAID
therapy at this time because this patient’s pain from
the acute flare has resolved, and he is not currently
experiencing an acute attack (Answer B is incorrect).
Pegloticase is reserved for patients with a severe gout
burden whose disease is refractory to or intolerant of
other ULTs (Answer D is incorrect).
Questions 1 and 2 pertain to the following case.
An 85-year-old woman (weight 65 kg) who resides at home with her daughter has a medical history significant for
type 2 diabetes and hypertension, and 1 year ago, she had a right hip fracture after a fall. Her regularly scheduled
medications include glyburide 10 mg daily, lisinopril 10 mg daily, metformin 500 mg twice daily, aspirin 81 mg
daily, and a multivitamin daily. Her as-needed medications include melatonin 6 mg at bedtime as needed for sleep,
meclizine 25 mg ½ tablet three times daily as needed for dizziness, and docusate 100 mg twice daily. Her laboratory
results show fasting plasma glucose 90 mg/dL, sodium (Na) 138 mEq/L, potassium (K) 4.5 mEq/L, chloride
(Cl) 102 mEq/L, carbon dioxide (CO2) 25 mEq/L, blood urea nitrogen (BUN) 30 mg/dL, SCr 1.8 mg/dL, and TSH
4.0 mU/L.
1. Considering the potential for altered pharmacokinetics, which set of medications is most likely to cause problems
for the patient?
A. Aspirin and melatonin.
B. Lisinopril and meclizine.
C. Lisinopril and metformin.
D. Glyburide and metformin.
- Answer: D
Renal elimination is usually the most significantly
changed pharmacokinetic value in older adults. This
patient’s advanced age and diseases will add to her loss
of renal function. Using the Cockcroft-Gault equation,
this patient’s estimated CrCl is 24 mL/minute/1.73 m2.
Creatinine clearance = [(140 − 85) × 65]/[(72 × 1.8)] ×
0.85. At this level of function, glyburide elimination
would be prolonged, and metformin use is contraindicated
(Answer D is correct). The aspirin is low dose,
and melatonin is safe even at very high doses (Answer
A is incorrect). Although lisinopril is renally eliminated,
dosing is based on response, and meclizine has
mostly hepatic metabolism with no dosage adjustment
in renal insufficiency (Answer B is incorrect). Answer
C is incorrect because lisinopril, unlike glyburide, is
not considered potentially inappropriate in older adults.
Questions 1 and 2 pertain to the following case.
An 85-year-old woman (weight 65 kg) who resides at home with her daughter has a medical history significant for
type 2 diabetes and hypertension, and 1 year ago, she had a right hip fracture after a fall. Her regularly scheduled
medications include glyburide 10 mg daily, lisinopril 10 mg daily, metformin 500 mg twice daily, aspirin 81 mg
daily, and a multivitamin daily. Her as-needed medications include melatonin 6 mg at bedtime as needed for sleep,
meclizine 25 mg ½ tablet three times daily as needed for dizziness, and docusate 100 mg twice daily. Her laboratory
results show fasting plasma glucose 90 mg/dL, sodium (Na) 138 mEq/L, potassium (K) 4.5 mEq/L, chloride
(Cl) 102 mEq/L, carbon dioxide (CO2) 25 mEq/L, blood urea nitrogen (BUN) 30 mg/dL, SCr 1.8 mg/dL, and TSH
4.0 mU/L.
- Considering the potential for increased pharmacodynamic sensitivity, which set of medications is most likely
to cause problems for the patient?
A. Aspirin and melatonin.
B. Lisinopril and meclizine.
C. Lisinopril and metformin.
D. Glyburide and metformin.
- Answer: B
Common pharmacodynamic changes associated with
aging include impaired homeostasis for electrolytes
with angiotensin-converting enzyme inhibitors such as
lisinopril and increased sensitivity to anticholinergic
adverse effects from drugs such as meclizine (Answer
B is correct). Lisinopril, metformin, and glyburide have
primarily pharmacokinetic problems because of renal
excretion changes when used in older adults (Answers C
and D are incorrect). Melatonin is extremely safe without
pharmacodynamic or pharmacokinetic issues in
older adults, and the aspirin is low dose, so issues with
GI bleeding are less than with higher doses (Answer A
is incorrect).
Questions 3–5 pertain to the following case.
A 70-year-old woman (height 66 inches, weight 71.7 kg [158 lb]) is in the clinic for an evaluation by the clinical pharmacist
for polypharmacy. She has complaints of fatigue, light-headedness, constipation, and “too many medicines.”
Her medical history is significant for hypertension, coronary artery disease (drug-eluting stent 8 years ago), chronic
obstructive pulmonary disease, diabetes mellitus, incontinence, frequent urinary tract infections, depression, and
moderate dementia. Vital signs include blood pressure 160/82 mm Hg, heart rate 51 beats/minute, respiratory rate
16 breaths/minute, and oxygen saturation 99% on room air. Her current medications are as follows: fluticasone/salmeterol
250/50 1 puff twice daily, aspirin 81 mg daily, acetaminophen 650 mg three times daily, clopidogrel 75 mg
daily, donepezil 10 mg daily, glipizide 5 mg twice daily, lisinopril 10 mg daily, loratadine 10 mg daily, metoprolol 50
mg twice daily, paroxetine 40 mg daily, ranitidine 150 mg twice daily, simvastatin 40 mg at bedtime, and tolterodine
2 mg at bedtime. Nitrofurantoin 100 mg twice daily for 10 days was initiated 3 days ago. Laboratory values from
her physician visit 3 days before are as follows: Na 130 mg/dL, K 4.2 mEq/dL, Cl 99 mg/dL, CO2 24 mEq/dL, BUN
24 mg/dL, SCr 1.6 mg/dL, fasting glucose 67 mg/dL, A1C 6.3%, urinalysis unremarkable except for blood- small,
pH 7.5, RBC 11–25/high-power field (HPF), white blood cells 0–2/HPF, and bacteria 168/HPF.
3. Which medication list best depicts the medications with the greatest potential to harm this patient, according
to the AGS 2019 Beers Criteria?
A. Paroxetine, donepezil, tolterodine.
B. Donepezil, glipizide, simvastatin.
C. Glipizide, donepezil, nitrofurantoin.
D. Metoprolol, clopidogrel, ranitidine.
- Answer: A
Glipizide and simvastatin are not listed in the 2019 AGS
Beers Criteria tables (Answer B is incorrect). In addition
to glipizide not being listed, the patient’s CrCl is
greater than 30 mL/minute/1.73 m2, and nitrofurantoin
is not for long-term suppression of bacteria (Answer C
is incorrect). Histamine-2 receptor antagonists are no
longer recommended to avoid in patients with dementia
or cognitive impairment because of weak evidence of
adverse cognitive effects, and neither metoprolol nor
clopidogrel is listed (Answer D is incorrect). However,
paroxetine should be used with caution in patients with
hyponatremia; donepezil as a CI should be avoided in
patients with syncope because it can cause bradycardia; and tolterodine has strong anticholinergic properties
and should be avoided in patients with dementia
(Answer A is correct).